0
$\begingroup$

I am having trouble with an integral:

$$\frac2L\int_L^\infty C\sin\left(\frac{n\pi x}{L}\right) dx$$ Where $C$ is just a constant.

I can't see how to do this, despite it apparently being rather straight forward. Something isn't making sense.

$$\frac{2C}L\int_L^\infty \sin\left(\frac{n\pi x}{L}\right)dx$$ $$\frac{2C}L\left[-\frac{L}{n\pi}\cos\left(\frac{n\pi x}{L}\right)\right]_L^\infty$$

But already I can't see how I can compute a divergence $\cos$.

Thank you for listening.

$\endgroup$
4
  • $\begingroup$ What is $u_0$ supposed to be? $\endgroup$ – David H Oct 19 '14 at 11:31
  • $\begingroup$ @DavidH Sorry David, that is just a constant $\endgroup$ – Analysis Oct 19 '14 at 11:32
  • $\begingroup$ What is the context of this integral? Because that looks related to the calculation of Fourier sine series coefficients. $\endgroup$ – PolandAspect Oct 19 '14 at 11:44
  • $\begingroup$ @PolandAspect Yes I think I configured it wrong, cont here math.stackexchange.com/questions/980756/setting-up-fourier $\endgroup$ – Analysis Oct 19 '14 at 11:50
0
$\begingroup$

So our integral is:

$$\frac2L\int_L^\infty C\sin\left(\frac{n\pi x}{L}\right) dx$$

Using improper integrals:

$$\lim_{b\to\infty}\frac{2}{L}\int_L^b C\sin\left(\frac{n\pi x}{L}\right) dx$$

$$\lim_{b\to\infty}\frac{2}{L} \left[C\sin\left(\frac{n\pi x}{L}\right)\right]_L^b dx$$

$$\lim_{b\to\infty}\frac{2}{L}\left(C\sin\left(\frac{n\pi b}{L}\right) - C\sin\left(\frac{n\pi L}{L}\right)\right)$$

Since

$$\lim_{b\to\infty}C\sin\left(\frac{n\pi b}{L}\right) = \infty$$

The integral diverges.

$\endgroup$
5
  • $\begingroup$ I understand. I was simply trying to show it diverged $\endgroup$ – Varun Iyer Oct 19 '14 at 11:47
  • $\begingroup$ Sorry I should have known that, I wasn't trying to be rude $\endgroup$ – Analysis Oct 19 '14 at 11:48
  • $\begingroup$ Its fine. You weren't trying to be rude. $\endgroup$ – Varun Iyer Oct 19 '14 at 11:48
  • $\begingroup$ It diverges, but not to infinity, so the last equality isn't correct. $\endgroup$ – Hans Lundmark Oct 19 '14 at 12:23
  • $\begingroup$ Sorry yeah, I posted that and deleted it since the way I phrased it was rude $\endgroup$ – Analysis Oct 19 '14 at 12:24
0
$\begingroup$

The integral is divergent (as your computation shows).

$\endgroup$
1

Your Answer

By clicking “Post Your Answer”, you agree to our terms of service, privacy policy and cookie policy

Not the answer you're looking for? Browse other questions tagged or ask your own question.